Equiangularity of a Polar Equation

  • Thread starter Thread starter daisukekazama
  • Start date Start date
  • Tags Tags
    Polar
Click For Summary
The discussion focuses on proving that a given polar equation connects two points while maintaining a constant angle with rays of constant theta. The user has identified that the equation, when projected onto a sphere, describes a loxodrome and aims to demonstrate that it is an equiangular spiral. They express frustration over the complexity of calculating the derivative dr/dθ, seeking a more elegant solution than brute force. A suggestion is made to simplify the derivative by extracting constants, which can make the calculation easier. The conversation emphasizes the challenge of deriving the equation while maintaining clarity in the solution process.
daisukekazama
Messages
2
Reaction score
0

Homework Statement


Show that the equation below connects the point (r_{0}, \theta_{0}) to the point (r_{1}, \theta_{1}), \theta_{0}\neq\theta_{1}, along a curve that everywhere forms the same angle with the rays \theta=constant.

And here's the equation. I can't get the Latex to work... no clue why.

Untitled-1-2.jpg


Homework Equations


N/A.

The Attempt at a Solution


This is part of a larger problem on stereographic projection - I've figured out that this equation, if projected onto a sphere, traces out a loxodrome. I also know that I essentially have to prove that that mess of an equation is a equiangular spiral (logarithmic spiral).

The problem is, no matter how I look at it, I need to calculate the derivative dr/d\theta, which - from where I'm looking - looks like one hell of a messy and convoluted derivative, which I'd rather not do.

Is there an easier, cleaner, more elegant way of solving this problem than brute-forcing the derivative? If not, how do I do the derivative?
 
Physics news on Phys.org
I'm not sure whether you have addressed this part of the question or not, but substitution shows that r indeed connects the given points. I'm not sure of a more elegant way at the moment, but the derivative is much easier to calculate if you extract constants in the way r^(x-b) = r^x/r^b.

Then you are left simply in the form r= C r_0^{t\theta} r_1^{-t\theta} and apply the product rule.
 
Thank you - that helped.
 
Question: A clock's minute hand has length 4 and its hour hand has length 3. What is the distance between the tips at the moment when it is increasing most rapidly?(Putnam Exam Question) Answer: Making assumption that both the hands moves at constant angular velocities, the answer is ## \sqrt{7} .## But don't you think this assumption is somewhat doubtful and wrong?

Similar threads

  • · Replies 11 ·
Replies
11
Views
3K
  • · Replies 5 ·
Replies
5
Views
3K
  • · Replies 2 ·
Replies
2
Views
2K
  • · Replies 2 ·
Replies
2
Views
5K
Replies
1
Views
2K
Replies
7
Views
2K
  • · Replies 4 ·
Replies
4
Views
2K
  • · Replies 6 ·
Replies
6
Views
2K
  • · Replies 2 ·
Replies
2
Views
3K
  • · Replies 14 ·
Replies
14
Views
3K